Momento angular con respecto al centro de masa.

Me han dicho [ Advertencia : dejo esto porque es lo que pedí y permite entender los diálogos en los comentarios, pero Azad, a quien agradezco, ha señalado que la fórmula no se cumple en general en la forma en que se expresa] que El momento angular y el cuerpo rígido con respecto a cualquier punto PAG PAG siempre se puede expresar como

L PAG = r c m × M v c m + ( yo metro yo R 2 yo ) ω L PAG = r C metro × METRO v C metro + ( yo metro yo R yo 2 ) ω
dónde r c m r C metro es la posición del centro de masa con respecto a PAG PAG , METRO METRO la masa del cuerpo R yo R yo la distancia de la yo yo -ésimo punto, teniendo masa metro yo metro yo , componiendo el cuerpo, y yo metro yo R 2 yo = Yo yo metro yo R yo 2 = yo su momento de inercia con respecto al eje instantáneo de la rotación alrededor del centro de masa de velocidad angular ω ω .

Yo se que la velocidad v yo v yo de cada punto PAG yo PAG yo teniendo masa metro yo metro yo , de un cuerpo rígido de masa METRO METRO puede verse como la suma de una velocidad de traslación de uno de sus puntos C C más una velocidad de rotación alrededor de ese punto: v yo = v C + ω × C PAG yo - - v yo = v C + ω × C PAG yo . Si elegimos C C como el centro de masa veo que

L c m = yo C PAG yo - - × m yo v yo = yo C PAG yo - - × m yo v c m + yo C PAG yo - - × m yo ( ω × C PAG yo - - ) L C metro = yo C PAG yo × metro yo v yo = yo C PAG yo × metro yo v C metro + yo C PAG yo × metro yo ( ω × C PAG yo )
= yo C PAG yo - - × m yo ( ω × C PAG yo - - ) = yo C PAG yo × metro yo ( ω × C PAG yo )
porque si no me equivoco yo C PAG yo - - × m yo v C = ( yo metro yo C PAG yo - - ) × v C = 0 yo C PAG yo × metro yo v C = ( yo metro yo C PAG yo ) × v C = 0 0 ya que yo metro yo C PAG yo - - yo metro yo C PAG yo es la posición del centro de masa con respecto a sí mismo, que es 0 0 0 0 .

¿Cómo se puede probar que yo C PAG yo - - × m yo ( ω × C PAG yo - - ) = ( yo metro yo R 2 yo ) ω yo C PAG yo × metro yo ( ω × C PAG yo ) = ( yo metro yo R yo 2 ) ω ? He buscado mucho en Internet y en libros, pero no encuentro nada. Para dar algunos antecedentes míos, no he estudiado nada de mecánica analítica. La fórmula me parece muy, muy interesante tanto en sí misma como porque, si el momento de inercia no depende del tiempo, t yo ( t ) = I ( t 0 0 ) t yo ( t ) = yo ( t 0 0 ) , la expresión anterior se puede diferenciar para obtener la fórmula del par resultante con respecto al centro de masa τ c m = d L c m re t = Yo α c m τ C metro = re L C metro re t = yo α C metro dónde α α es la aceleración angular alrededor del centro de masa. Muchas gracias por cualquier respuesta!


Algunas pruebas infructuosas: al usar la "identidad BAC CAB" como lo sugirió Azad, a quien agradezco sinceramente, a × ( b × c ) = ( a c ) b - ( a b ) c una × ( si × C ) = ( una C ) si - ( una si ) C , Puedo ver eso

yo C PAG yo - - × m yo ( ω × C PAG yo - - ) = yo metro yo C PAG yo - - 2 ω - m yo ( C PAG yo - - ω ) C PAG yo - - yo C PAG yo × metro yo ( ω × C PAG yo ) = yo metro yo C PAG yo 2 ω - metro yo ( C PAG yo ω ) C PAG yo
que, al descomponer cada C PAG yo - - C PAG yo en un componente axial UNA yo UNA yo y un componente radial R yo R yo , cuyas normas respectivamente son UNA yo UNA yo y R yo R yo , con R yo R yo como la distancia de yo yo al eje de rotación, se convierte
yo metro yo R 2 yo ω + yo metro yo UNA 2 yo ω - m yo ( A yo ω ) C PAG yo - - yo metro yo R yo 2 ω + yo metro yo UNA yo 2 ω - metro yo ( UNA yo ω ) C PAG yo
pero no puedo probar eso yo metro yo UNA 2 yo ω - m yo ( A yo ω ) C PAG yo - - = 0 yo metro yo UNA yo 2 ω - metro yo ( UNA yo ω ) C PAG yo = 0 0 .

¿No es esto solo una consecuencia del teorema del eje paralelo? Las pruebas del teorema deberían ser fáciles para Google.
Su primera ecuación describe el momento angular total alrededor del origen y usted está a solo un BAC CAB lejos de la prueba
No estoy seguro de que llamaría a esto una prueba en ningún sentido de la palabra. El cálculo simplemente conduce a la derivación de yo yo , es decir, es la definición de momentos de inercia más que cualquier otra cosa.
Considero que es un mal hábito introducir cosas en la física que uno "sabe a priori" solo para tener la sensación de "probar algo". Su conocimiento a priori es una derivación trivial de la misma expresión que está utilizando en su "prueba". La forma mucho mejor es tratar todo como una derivación. Funciona igual de bien y no choca con la definición de ciencia, que no es lo mismo que la definición de matemáticas.
Le recomiendo que lea el primer capítulo de Mecánica clásica de Goldstein. Tiene una buena y breve encuesta de estas cosas.

Respuestas (2)

Creo que estás complicando demasiado esto. Considere un punto arbitrario P que se mueve con velocidad lineal v UNA v UNA .

  • El impulso lineal es
    P = m v c m PAG = metro v C metro
  • El momento angular en el centro de masa es
    L c m = Yo c m ω L C metro = yo C metro ω
  • La velocidad lineal del centro de masa es
    v c m = v UNA + ω × r c m v C metro = v UNA + ω × r C metro
    dónde r c m r C metro es la ubicación del centro de masa en relación con A.
  • El momento lineal en términos del movimiento de A es
    P = m ( v UNA + ω × r c m ) PAG = metro ( v UNA + ω × r C metro )
    P = m v UNA - m r c m × ω PAG = metro v UNA - metro r C metro × ω
  • El momento angular en A es
    L UNA = L c m + r c m × P L UNA = L C metro + r C metro × PAG
    que se expande como
    L UNA = Yo c m ω + r c m × m v c m = Yo c m ω + r c m × m ( v UNA + ω × r c m ) L UNA = yo C metro ω + r C metro × metro v C metro = yo C metro ω + r C metro × metro ( v UNA + ω × r C metro )

L UNA = Yo c m ω - m r c m × r c m × ω + m v UNA L UNA = yo C metro ω - metro r C metro × r C metro × ω + metro v UNA

  • El momento espacial combinado en A produce la matriz de inercia espacial 6 × 6 en A

^ UNA = Yo UNA v ^ UNA ^ UNA = yo UNA v ^ UNA
{ P L UNA } = [ m m [ r c m × ] - m [ r c m × ] yo c m - m [ r c m × ] [ r c m × ] ] { v UNA ω } { PAG L UNA } = [ metro - metro [ r C metro × ] metro [ r C metro × ] yo C metro - metro [ r C metro × ] [ r C metro × ] ] { v UNA ω }

NOTA: para los extraños [ r × ] [ r × ] Notación que parece faltar un vector. ¿Qué es el Vector / Producto Cruzado?

  • El momento de inercia de masa en A se define así como
    yo UNA = Yo c m - m [ r c m × ] [ r c m × ] yo UNA = yo C metro - metro [ r C metro × ] [ r C metro × ]
    Esta es una representación vectorial del teorema del eje paralelo .
  • Finalmente, necesita diferenciar las expresiones de impulso para llegar a las 6 ecuaciones de movimiento de Newton-Euler (consulte https://physics.stackexchange.com/a/80449/392 )
¡Muchas gracias! Perdóname: no entiendo L UNA = Yo c m ω - m r c m × r c m × ω + m v UNA L UNA = yo C metro ω - metro r C metro × r C metro × ω + metro v UNA . En cuanto a yo c m yo C metro , ¿es una matriz? La única definición que conozco, utilizada por mi libro, La física de Gettys-Keller-Skove, de momento de inercia yo yo es un escalar yo : = yo metro yo R 2 yo yo : = yo metro yo R yo 2 -y yo = V ρ R 2 re V yo = V ρ R 2 re V para un cuerpo continuo donde R R es la distancia del punto desde el eje.
yo c m yo C metro es el momento de masa del tensor de inercia. Se define como
yo c m = yo x x yo x y yo x z yo x y yo y y yo y z yo x z yo y z yo z z yo C metro = ( yo X X yo X y yo X z yo X y yo y y yo y z yo X z yo y z yo z z )
(Ver farside.ph.utexas.edu/teaching/336k/Newtonhtml/node64.html ). Contiene los componentes de la inercia para cada eje en diagonal y los términos cruzados en la diagonal desactivada. A medida que el cuerpo gira (con una matriz de rotación 3 × 3 mi mi ) los componentes de yo c m yo C metro cambio también. Esto se hace con
yo c m = E yo b o d y mi yo C metro = mi yo si o re y mi
.
Muy, muy interesante ¡No puedo esperar para estudiarlo! ¡De nuevo, muchas gracias!

Si nos fijamos en C PAG yo X metro yo ( ω × C PAG yo ) C PAG yo X metro yo ( ω × C PAG yo ) podemos decir que el producto cruzado en el paréntesis da el componente del vector C PAG yo C PAG yo a lo largo de la dirección de ω ω . Llamemos a ese componente R yo R yo . ( nota : R yo R yo es la distancia perpendicular entre la partícula en el sistema de partículas en el que estamos interesados ​​y el eje de rotación del sistema de partículas)

Ahora tenemos:

C PAG yo × m yo R yo ω C PAG yo × metro yo R yo ω = metro yo R 2 yo ω metro yo R yo 2 ω

(Estamos tomando el producto cruzado de C PAG yo C PAG yo con R yo R yo que está en la dirección perpendicular a ambos ω ω y C PAG yo C PAG yo que nuevamente nos dará R yo R yo . )

Así tenemos:

L c m = ( yo metro yo R 2 yo ) ω L C metro = ( yo metro yo R yo 2 ) ω

Oh! lo siento. Mi error. He hecho la corrección en la respuesta.
Su pregunta me hizo pensar en la definición básica de la dirección de ω ω . Siempre se define con respecto a los vectores de posición. r r y r + Δ r r + Δ r dónde: Δ r = r Δ θ θ ^ Δ r = r Δ θ θ ^ , θ θ siendo el ángulo atravesado por el vector r r a tiempo Δ t Δ t y θ ^ θ ^ siendo la dirección tangente a la trayectoria de movimiento de la partícula (que es circular). Estoy realmente agradecido con usted @ Self-teachingDavide por hacer esta pregunta que condujo a la definición fundamental de dirección de ω ω .
Volviendo a la pregunta, se puede ver que para tomar el producto cruzado ω × r ω × r , la dirección de ω ω no se puede tomar como k ^ k ^ . Para obtener la dirección de la velocidad angular, uno debe conocer el camino de la partícula, obtener el vector unitario de la tangente ( θ ^ θ ^ ) a r r y encuentra el producto cruzado: r ^ × θ ^ r ^ × θ ^ . Por favor, corríjame si estoy equivocado.
Las matemáticas son bastante difíciles y no he visto tal prueba presentada en ningún lado. Todos mis cursos universitarios trataron problemas 2D donde r r siempre estaba en el plano XY y por lo tanto ω ω siempre estuvo en el k ^ k ^ dirección. Además, la mayoría de los libros presentan la prueba solo para cuerpos rígidos planos y mencionan que lo mismo es aplicable a todos los cuerpos rígidos.
Lamento haber puesto tantos comentarios. Consulte este enlace para obtener una descripción completa de la velocidad angular a través de los ángulos de Euler: physics.stackexchange.com/questions/73961/…